Đến nội dung

ninhxa nội dung

Có 138 mục bởi ninhxa (Tìm giới hạn từ 09-05-2020)



Sắp theo                Sắp xếp  

#339138 Topic bất đẳng thức THCS (2)

Đã gửi bởi ninhxa on 23-07-2012 - 09:42 trong Bất đẳng thức và cực trị

Bài 471. Cho $0\leq a,~b,~c\leq 1$. CMR: $a^2+b^2+c^2\leq 1+a^2b+b^2c+c^2a $

-Do $0\leq a,b,c\leq 1\to \left\{\begin{matrix}abc\geq 0 \\ a\geq a^2 \\ b\geq b^2 \\ c\geq c^2 \end{matrix}\right.$
-Do đó:
$VP\geq a^2b^2+b^2c^2+c^2a^2+1-abc$
-Lại có:
$a^2b^2+b^2c^2+c^2a^2+1-abc-a^2-b^2-c^2=(1-a^2)(1-b^2)(1-c^2)\geq 0$
(do $0\leq a,b,c\leq 1$)
$\rightarrow dpcm$



#337888 Topic bất đẳng thức THCS (2)

Đã gửi bởi ninhxa on 19-07-2012 - 22:40 trong Bất đẳng thức và cực trị

Bài 467: Co 3 số thực x,y,z thoả mãn x(x-1)+ y(y-1) +z(z-1)$\leq \frac{4}{3}$. Chứng minh: x+y+z$\leq 4$

-Áp dụng bất đẳng thức Cauchy Shwarz ta dc: $3(x^2+y^2+z^2)\geq (x+y+z)^2$$\Rightarrow \frac{\left ( x+y+z \right )^2}{3}-(x+y+z)\leq x(x-1)+y(y-1)+z(z-1)\leq 4$
$\Rightarrow \left [ \left ( x+y+z \right ) -4\right ]\left [ \left ( x+y+z \right )+1 \right ]\leq 0$
$\rightarrow dpcm$



#318100 Topic bất đẳng thức THCS (2)

Đã gửi bởi ninhxa on 20-05-2012 - 15:43 trong Bất đẳng thức và cực trị

BÀI 360:
Cho a,b,c>0. Chứng minh rằng:
$\frac{{{a^3}}}{{{{\left( {b + c} \right)}^2}}} + \frac{{{b^3}}}{{{{\left( {c + a} \right)}^2}}} + \frac{{{c^3}}}{{{{\left( {a + b} \right)}^2}}} \ge \frac{{a + b + c}}{4}$
-------
P/S: Topic này dần lặng đi rồi, mọi người tiếp tục post bài nào !
-------


-Áp dụng bất đẳng thức AM-GM ta có

$\frac{a^3}{(b+c)^2}+\frac{b+c}{8}+\frac{b+c}{8}\geq \frac{3a}{4}$

-Xây dựng các bất đẳng thức tương tự rồi cộng vế với vế ta được điều cần chứng minh.



#316546 Topic bất đẳng thức THCS (2)

Đã gửi bởi ninhxa on 14-05-2012 - 21:00 trong Bất đẳng thức và cực trị

Bài 359: CMR với mọi số thực dương $a,b,c$ thì:
$$\frac{a^3}{2a^2-ab+2b^2}+\frac{b^3}{2b^2-bc+2c^2}+\frac{c^3}{2c^2-ca+2b^2}\geq \frac{a+b+c}{3}$$


-Áp dụng bdt AM_GM ta có $2a^2+2b^2-ab\geq 4ab-ab=3ab$

-Ta có
$$\frac{a^3}{2a^2-ab+2b^2}=\frac{1}{2}.\frac{a(2a^2-ab+2b^2)-ab(2b-a)}{2a^2-ab+2b^2}\geq \frac{1}{2}\left ( a-\frac{ab(2b-a)}{3ab} \right )\geq \frac{1}{2}\left ( a-\frac{2b-a}{3} \right )$$


-Xây dựng các bất đẳng thức tượng tự rồi cộng vào ta được điều cần chứng minh.



#348003 Topic bất đẳng thức THCS (2)

Đã gửi bởi ninhxa on 18-08-2012 - 20:53 trong Bất đẳng thức và cực trị

Bài 494
Bài làm
a, ta có :$ a^2 +b^2 \leq (a+b)^2 =4$
$ab \leq \frac{(a+b)^2}{4}=1$
$\Rightarrow ab(a^2+b^2) \leq 4 $
Vậy$ A_{Max} =4$
Dấu $=$ sảy ra $\leftrightarrow a=b=0$

-Bất đẳng thức đó là j` vậy.
-Bài này làm như sau:
Áp dụng bdt am-gm ta có:
$2ab(a^2+b^2)\leq \frac{(a^2+b^2+2ab)^2}{4}=\frac{(a+b)^4}{4}=4$



#303000 Topic bất đẳng thức THCS (2)

Đã gửi bởi ninhxa on 08-03-2012 - 21:59 trong Bất đẳng thức và cực trị

Lúc mới vào thì ghiền cái topic này nhất mà sao giờ chán thế
291 Cho $a,b,c$ là độ dài 3 cạnh của tam giác
CMR : $\frac{a^4}{b+c} + \frac{b^4}{c+a} +\frac{c^4}{a+b} < 2(a^2b+b^2c+c^2a)$


Giả sử $a\geq b\geq c$
$\Leftrightarrow (a-c)(b-c)(a-b)\geq 0$
$\Leftrightarrow a^{2}b+b^{2}c+c^{2}a \geq a^{2}c+b^{2}a+c^{2}b$
$\Leftrightarrow 2(a^{2}b+b^{2}c+c^{2}a)\geq a^{2}b+b^{2}c+c^{2}a + a^{2}c+b^{2}a+c^{2}b=a^{2}(b+c)+b^{2}(c+a)+c^{2}(a+b)> a^{3}+b^{3}+c^{3}$
$>\frac{a^4}{b+c} + \frac{b^4}{c+a} +\frac{c^4}{a+b}$



#328825 Topic luyện thi vào lớp 10 năm 2013 – 2014 (Hình học)

Đã gửi bởi ninhxa on 24-06-2012 - 21:34 trong Hình học

Bài 128. Cho (O;R) và dây AB<2R. Tiếp tuyến tại A và B của (O) cắt nhau tại M. Gọi T là trung điểm MA và K là giao điểm của BT và (O). Gọi H là giao điểm của AB và MO, F là giao điểm của KH và (O).
a)Chứng minh rằng: MO là tia phân giác của góc KMF
b)Gọi C là giao điểm thứ 2 của MK với (O). Chứng minh tam giác ABC cân tại A



#346440 Topic hình học THCS

Đã gửi bởi ninhxa on 13-08-2012 - 13:23 trong Hình học

Bài 98: Cho tam giác $ABC$ nội tiếp đường tròn $(O)$. $AD$ là đường kính của $(O)$. $M$ là trung điểm của $BC$, $H$ là trực tâm tam giác $ABC$. Gọi $X$, $Y$, $Z$ lần lượt là hình chiếu của $D$ lên $HB$, $HC$, $BC$. Chứng minh rằng $X$, $Y$, $Z$, $M$ cùng thuộc một đường tròn.
----

-Bài này cho nhiều giả thiết để gây nhiễu.
Dễ thấy: tứ giác HBDC là hình bình hành. Đến đây, tớ chỉ vẽ tứ giác này cho hình nó to :D
Hình đã gửi
-Vì tứ giác HBDC là hình bình hành và M là trung điểm BC nên M là trung điểm HD
-Xét tam giác HXD vuông ở X có MH=MD nên:
$XM=HM\Rightarrow \widehat{HMX}=180^0-2.\widehat{XHM}$
-Tg tự $\widehat{XMY}=180^0-2\widehat{YHM}$
-Do đó: $\widehat{XMY}=\widehat{HMX}+\widehat{HMY}=360^0-2(\widehat{XHM}+\widehat{MHY})=2(180^0-\widehat{BHC})=2.\widehat{HBD}$
-Dễ thấy: tứ giác XZDB và ZYCD nội tiếp
$\Rightarrow \widehat{XZY}=360^0-(\widehat{XZD}+\widehat{YZD})=\widehat{XBD}+\widehat{YCD}=2\widehat{HBD}$
-Vậy: $\widehat{XMY}=\widehat{XZY}$ nên tứ giác XMZY nội tiếp
$\Rightarrow dpcm$



#339492 Topic hình học THCS

Đã gửi bởi ninhxa on 24-07-2012 - 09:42 trong Hình học

______________________________
Bài 45: Cho $\triangle ABC$, $H$ là trực tâm.
CMR: $\frac{BH.CH}{AB.AC}+\frac{CH.AH}{BC.BA} + \frac{AH.BH}{AC.BC} =1$
p/s: bài không khó :)

Hình đã gửi
-Ta có
$\Delta CHE\sim \Delta BAE(g-g)$
$\to \frac{CH}{AB}=\frac{CE}{BE}$
$\to \frac{CH.BH}{AB.AC}=\frac{CE.BH}{BE.AC}=\frac{S_{BHC}}{S_{ABC}}$
-Xây dựng các đẳng thức tương tự ta được dpcm/



#337631 Tuyển tập một số bài phương trình, hệ phương trình thi HSG tỉnh

Đã gửi bởi ninhxa on 19-07-2012 - 14:09 trong Phương trình - hệ phương trình - bất phương trình

Bài 58. Giải hệ phương trình

$$\left\{\begin{matrix}
9y^3(3x^3-1)=-125 & & \\ 45x^2y+75x=6y^2
& &
\end{matrix}\right.$$

Đề thi HSG tỉnh Đồng Tháp 09/10


-Công nhận mình có duyên với đề đồng tháp thật
-Nhận thấy $y\neq 0$ nên hệ phương trình tương đương với:
$\left\{\begin{matrix}6y^3(3x^3-1)=\frac{-250}{3} \\45x^3y^3+75xy=6y^3 \end{matrix}\right.$
-Trừ vế với vế ta dc:
$18x^3y^3-45x^2y^2-75xy+\frac{250}{3}=0$
$\Leftrightarrow \begin{bmatrix}xy=\frac{-5}{3} \\ xy=\frac{10}{2} \\ xy=\frac{5}{6} \end{bmatrix}$
-Thay xy trên vào pt (1) ta tìm dc y sau đó là x.
-Hệ phương trình có các nghiệm là: $(x;y)=\left ( \frac{2}{3};5 \right );\left ( \frac{1}{3} ;\frac{5}{2}\right )$



#336831 Tuyển tập một số bài phương trình, hệ phương trình thi HSG tỉnh

Đã gửi bởi ninhxa on 17-07-2012 - 14:40 trong Phương trình - hệ phương trình - bất phương trình

Như vậy là đã xong bé 39,40. Còn 38 nữa thôi, post luôn bài mới :D


Bài 41. Giải hệ phương trình

$$\left\{\begin{matrix}
\sqrt{9+8x^2y-x^4y^2}=y\left ( 16y^5-3x^2y^2+1 \right ) \\ 1+\sqrt{16+\left ( x-2y \right )^2}=x^2\left ( 5y^3-x^2 \right )+y
\end{matrix}\right.$$

Đề chọn HSG lớp 12 dự thi cấp quốc gia - Đồng Tháp - 2011/2012

Chém gió: Bài này ý tưởng chắc là đánh giá, nhìn qua qua thấy quen mắt lắm ! :D

------------


-Bạn chém gió chuẩn thế :D
-Ta có
$y(16y^5-3x^2y^2+1)-[x^2(5y^3-x^2)+y]=(4y^3-x^2)^2\geq 0$
$\Rightarrow \sqrt{9+8x^2y-x^4y^2}\geq 1+\sqrt{16+(x-2y)^2}$
-Lại có:
$ \sqrt{9+8x^2y-x^4y^2}\leq1+\sqrt{16+(x-2y)^2}$
-Bất đẳng thức trên đúng do
$ \sqrt{9+8x^2y-x^4y^2}=\sqrt{( 9-x^2y)(x^2y+1)}\leq 5$ (theo AM-GM)
và $1+\sqrt{16+(x-2y)^2}\geq 5$
-Do đó: $\sqrt{9+8x^2y-x^4y^2}= 1+\sqrt{16+(x-2y)^2}=5$
$\Leftrightarrow \left\{\begin{matrix}x=2y \\x^2y=4 \end{matrix}\right. \Leftrightarrow \left\{\begin{matrix}x=2 \\ y=1 \end{matrix}\right.$



#338481 Tuyển tập một số bài phương trình, hệ phương trình thi HSG tỉnh

Đã gửi bởi ninhxa on 21-07-2012 - 16:10 trong Phương trình - hệ phương trình - bất phương trình

Post tiếp đẩy topic lên tí :D

Bài 64. Giải hệ phương trình

$$\left\{\begin{matrix}
x^2-y^2=0 & & \\
x+y^2+12\sqrt[3]{x^2y}=2010 & &
\end{matrix}\right.$$

Đề chọn đội tuyển 11 - Chuyên Lê Quý Đôn - Quảng Trị 2011-2012.

P/S: Chọn đội tuyển 30/4 mà không khó lắm :D

-Hix. Chắc bài này để tránh điểm liệt quá
-Hệ tương đương với:
$\left\{\begin{matrix}x=\pm y \\ x+y^2+12y=2010 \end{matrix}\right.$
-Với $x=y$ thì $y^2+13y-2010=0$
-Với $x=-y$ thì $y^2+11y-2010=0$

p/s: Nghiệm lẻ quá.



#314355 Topic ôn tập vào lớp 10

Đã gửi bởi ninhxa on 04-05-2012 - 20:34 trong Tài liệu - Đề thi

Bài 1. Giải các phương trình sau:

b) $(12x-1)(6x-1)(4x-1)(3x-1)=5$




Phương trình tương đương với:
$(12x-1)(12x-2)(12-3)(12-4)=5.2.3.4=120$ (*)

Đặt 12x-4=t$
(*)\Leftrightarrow t(t+1)(t+2)(t+3)=120$
$\Leftrightarrow (t^2+3t)(t^2+3t+2)=120$ (**)

Đặt $t^2+3t+1=s$

$(**)\Leftrightarrow (s-1)(s+1)=120\Leftrightarrow s^2-1=120\Leftrightarrow s^2=121\Leftrightarrow s=\pm 11$

Từ đây ta thay lần lượt sẽ được nghiệm



#347147 Topic yêu cầu tài liệu THPT

Đã gửi bởi ninhxa on 16-08-2012 - 10:33 trong Tài liệu tham khảo khác

Cho mình xin tài liệu về định lý con nhím và ứng dụng của nó,thật sự mình search mãi mà không down được,ai cho mình với(mình muốn down về tiện cho việc học )

-Mình có tài liệu này

File gửi kèm




#349903 [MO2013] Trận 1 - Phương trình, hệ phương trình, bất phương trình

Đã gửi bởi ninhxa on 26-08-2012 - 15:52 trong Thi giải toán Marathon dành cho học sinh Chuyên Toán 2013

Đề thi trận 1
Giải hệ phương trình trên tập hợp số thực :
$$\begin{cases}
& \text \sqrt[8]{2.\sqrt[5]{7} - \sqrt[10]{y}} + (17 - \sqrt{37}).z^2 = 544 - 32.\sqrt{37} (1) \\
& \text x.(9.\sqrt{1 + x^2} + 13.\sqrt{1 - x^2}) + 4\sqrt{y} = 912 (2)\\
& \text \sqrt{(10.\sqrt{5} + 20).x.(1 - x)} + z.\sqrt[6]{8} = 10 (3)
\end{cases}$$

-ĐKXD:
$\left\{\begin{matrix}y\leq 50176 \\ 1-x^2\geq 0 \\ x(1-x)\geq 0 \end{matrix}\right.$
-Ta có:
*Xét $0\leq x$ (?) thì từ pt (2) ta có: $4\sqrt{y}\geq 912\Leftrightarrow y\geq 51984$ (trái dkxd)
*Xét $x>0$ ta có:
$x.(9.\sqrt{1 + x^2} + 13.\sqrt{1 - x^2})=9\sqrt{x^2+x^4}+13\sqrt{x^2-x^4}$
-Áp dụng bdt Cauchy-Shwarz và bdt phụ $ab\leq \frac{(a+b)^2}{4}$ (với mọi ab) ta có:
$\left ( 9\sqrt{x^2+x^4}+13\sqrt{x^2-x^4} \right )^2=\left ( 3\sqrt{3}.\sqrt{3x^2+3x^4}+\sqrt{13}.\sqrt{13x^2-13x^4} \right )^2\leq (13+27)(3x^2+3x^4+13x^2-13x^4)=16.5x^2(8-5x^2)\leq 16.\frac{(5x^2+8-5x^2)^2}{4}=16^2$
-Do đó:
$x.(9.\sqrt{1 + x^2} + 13.\sqrt{1 - x^2})\leq 16$
Dấu bằng xảy ra khi: $x=\frac{2}{\sqrt{5}}$
-Kết hợp với pt (2) ta có: $4\sqrt{y}\geq 896\Rightarrow y\geq 50176$
mà theo dkxd thì $y\leq 50176$ nên y=50176
-Từ đó: $x.(9.\sqrt{1 + x^2} + 13.\sqrt{1 - x^2})= 16$
$\Leftrightarrow x=\frac{2}{\sqrt{5}}$
-Thay y=50176 vào pt (1) ta tìm dc $z=\pm \sqrt{32}$
-Thử lại ta thấy $\left ( x;y;z \right )=\left ( \frac{2}{\sqrt{5}};50176;\sqrt{32} \right )$ là nghiệm của hpt
-Do vậy hệ có duy nhất 1 nghiệm là: $\left ( x;y;z \right )=\left ( \frac{2}{\sqrt{5}};50176;\sqrt{32} \right )$

p/s: ở cuộc thi trên diễn đàn pt, hệ pt toàn dùng bdt. liệu chủ đề này có phải là chủ đề bdt 2.0 ko nhỉ? Trá hình thôi.

Điểm bài: 10
S=48−(63−20)+3×10+0+0=35



#318318 Đăng kí tham gia Marathon for Secondary school 2012

Đã gửi bởi ninhxa on 21-05-2012 - 19:33 trong Thi giải toán Marathon cấp THCS 2012

Em đăng kí nhé.



#322989 Đề thi tuyển sinh chuyên SPHN (ngày 1)

Đã gửi bởi ninhxa on 06-06-2012 - 22:56 trong Tài liệu - Đề thi

Câu 5 (1 điểm). Cho các số thực dương x, y thỏa mãn điều kiện $\sqrt{xy}(x-y)=x+y$. Tìm GTNN của biểu thức $P=x+y$.


Có thể theo cách này: Ngẫu nhiên lại ra :lol:
Bình phương 2 vế của điều kiện ta có
$xy(x-y)^2=(x+y)^2$
$\Leftrightarrow xy\left [ (x+y)^2-4xy \right ]=(x+y)^2$
$\Leftrightarrow 4(xy)^2-(xy)(x+y)^2+(x+y)^2=0$
Đây là phương trình bậc 2 ẩn xy tham số x+y
Xét $\Delta=(x+y)^2(x+y+4)(x+y-4)$
Do tồn tại x,y thỏa mãn yêu cầu nên pt có nghiệm hay delta >=0
$\Rightarrow x+y\geq 4$

P/s: liệu phần c bài hình trên có sai đề ko vậy nhỉ? m` vẽ hình thấy cứ sai thế nào í



#328150 Tuyển sinh 10: TOÁN CHUYÊN (TP.HCM)

Đã gửi bởi ninhxa on 22-06-2012 - 22:16 trong Tài liệu - Đề thi

Câu 5: (2 điểm)
Cho tứ giác nội tiếp ABCD có AC, BD vuông góc nhau tại H. Gọi M là điểm trên cạnh AB sao cho $AM=\frac{1}{3}AB$ và N là trung điểm HC. Chứng minh $DN\perp MH$


Hình đã gửi

-Ta làm ngược lại. Giả sử HK vuông góc với DN tại K và cắt AB tại M. Ta cần chứng minh $AM=\frac{AB}{3}$
-Điều đó tương đương với chứng minh: $\frac{2S_{BHM}}{2S_{MHA}}=2$
-Ta có:$\frac{S_{BHM}}{S_{DHK}}=\frac{HM.BH.sin\widehat{BHM}}{HD.HK.sin\widehat{DHK}}=\frac{HM.BH}{HD.HK}$
Mà $S_{DHK}=\frac{1}{2}HK.DK$
Suy ra: $2S_{BHM}=\frac{KD.BH.MH}{DH}$
-Chứng minh tg tự $2S_{AMH}=\frac{NK.MH.AH}{HN}$
-Suy ra: $\frac{2S_{BHM}}{2S_{AMH}}=\frac{KD.BH.HN}{DH.NK.AH}=\frac{KD.(BH.HD).HN}{DH^2.NK.AH}=\frac{KD.AH.HC.HN}{DK.DN.NK.AH}=\frac{HC.HN}{DN.NK}=\frac{2HN^2}{HN^2}=2\rightarrow dpcm$



#328251 Tuyển sinh 10: TOÁN CHUYÊN (TP.HCM)

Đã gửi bởi ninhxa on 23-06-2012 - 10:57 trong Tài liệu - Đề thi

Mình hiểu ý của bạn rồi nhưng B =>A đúng chắc gì A=>B cũng đúng


Nhưng qua HK vuông góc với DN chỉ đi qua 1 điểm trên AB. Vậy 2 điểm đó phải trùng nhau chứ.



#320083 Trận 15 - "MSS22 nth1235" VS ALL

Đã gửi bởi ninhxa on 27-05-2012 - 20:56 trong Thi giải toán Marathon cấp THCS 2012

-Với a,b,c,x,y,z là các số dương ta có:
$x=\frac{xyz}{yz}=\frac{ax+by+cz}{yz}=\frac{ax}{yz}+\frac{b}{z}+\frac{c}{y}> \frac{b}{z}+\frac{c}{y}$

-Tượng tự ta có:
$y> \frac{a}{z}+\frac{c}{x}$
$z> \frac{b}{x}+\frac{a}{y}$

-Cộng vế với vế các bất đẳng thức trên ta có:
$x+y+z> \frac{a+b}{z}+\frac{c+a}{y}+\frac{b+c}{x}$
Hay $2(x+y+z)> \frac{a+b}{z}+\frac{c+a}{y}+\frac{b+c}{x}+x+y+z$ (*)

-Mà theo bất đẳng thức AM_GM cho 2 số dương ta có:
$x+\frac{b+c}{x}\geq 2\sqrt{b+c}$
$y+\frac{c+a}{y}\geq 2\sqrt{c+a}$
$z+\frac{a+b}{z}\geq 2\sqrt{a+b}$
Hay $\frac{a+b}{z}+\frac{c+a}{y}+\frac{b+c}{x}+x+y+z \geq 2\left ( \sqrt{a+b}+\sqrt{b+c}+\sqrt{c+a} \right )$ (**)

-Từ (*) và (**) ta có; $x+y+z> \sqrt{a+b}+\sqrt{b+c}+\sqrt{c+a}$
-Vậy hệ phương trình vô nghiệm

Bài giải chính xác. Chuẩn ko cần chỉnh :)
D - B = 11h
E = 10
F = 0
S = 67



#428415 Sử dụng khai triển $Abel$ để chứng minh bất đẳng thức

Đã gửi bởi ninhxa on 17-06-2013 - 23:46 trong Chuyên đề toán THPT

Bài toán 4:Với $a\geq 3, a+b\geq 5, a+b+c\geq 6$, chứng minh rằng

$a^2+b^2+c^2\geq 14$

 

bạn namsub nói đúng rồi đó

bài này có thể làm như sau:

áp dụng bdt cauchy-schwaz ta dc:

$(a^2+b^2+c^2)(3^2+2^2+1^2)\geq (3a+2b+c)^2$

theo phép nhóm abel ta có:

$3a+2b+c=(3-2)a+(2-1)(a+b)+a+b+c\geq 3+5+6=14$

ta có dc dpcm




#425274 Sử dụng khai triển $Abel$ để chứng minh bất đẳng thức

Đã gửi bởi ninhxa on 09-06-2013 - 00:34 trong Chuyên đề toán THPT

Bài này có lẽ dễ nhất   :( 

$a^{2}+b^{2}+c^{2}=a.a+b.b+c.c=(a-b).a+(b-c).(a+b)+c.(a+b+c)\geq 3(a-b)+5(b-c)+6c=a+(a+b)+(a+b+c)\geq 3+5+6=14 \Rightarrow đpcm$

bạn ko thể có đánh giá đó dc. a-b và b-c chưa biết dấu mà




#327316 Đề thi tuyển sinh trường THPT chuyên Hải Dương 2012-2013

Đã gửi bởi ninhxa on 20-06-2012 - 16:22 trong Tài liệu - Đề thi

1, Giải phương trình $(x^2-4x+11)(x^4-8x^2+21)=35$

-Chém câu dễ :D
-Có $(x^2-4x+11)(x^4-8x^2+21)=\left [ \left ( x-2 \right )^2+7 \right ]\left [ \left ( x^2-4 \right )^2+5 \right ]\geq 35$
Dấu bằng xảy ra khi x=2
-Do đó pt có nghiệm x=2



#326115 KÌ THI TUYỂN SINH VÀO LỚP 10 THPT CHUYÊN HÀ TĨNH

Đã gửi bởi ninhxa on 16-06-2012 - 22:26 trong Tài liệu - Đề thi

Bạn trình bày rõ cách đánh giá vế kia được không? Mình chưa làm ra, xét Delta nữa!

ĐK: x>=1
Vế còn lại là
$\frac{1}{\sqrt[3]{(x+6)^3}+2\sqrt[3]{(x+6)}+4}+\frac{1}{\sqrt{x-1}+1}=x+2$
Xét $x\geq 1$ ta có
$VT\leq \frac{1}{\sqrt[3]{49}+\sqrt[3]{7}+4}+1< 3\leq VP$
Do đó vế kia vô nghiệm

P/s Ai làm bài bất đẳng thức đi.



#327823 Đề thi tuyển sinh lớp 10 THPT Chuyên Thái Bình 2012-2013

Đã gửi bởi ninhxa on 22-06-2012 - 05:40 trong Tài liệu - Đề thi

Lâu lâu mới thấy bài số :)
(False) SOLUTION

Theo giả thiết:
$$m+n-1|2(m^2+n^2)-1\\ \Leftrightarrow m+n-1| (m+n)^2+(m-n)^2-1\\ \Leftrightarrow m+n-1| (m+n-1)(m+n+1)+(m-n)^2\\ \Leftrightarrow m+n-1|(m-n)^2\ (1)$$
-Áp dụng t/c: Nếu số chính phương $a$ chia hết cho số nguyên tố $p$ thì $a$ cũng chia hết cho $p^2$, ta có:
$$(1)\Rightarrow (m+n-1)^2| (m-n)^2\\\Leftrightarrow m^2+n^2+1+2mn-2m-2n | m^2-2mn+n^2
\\\Rightarrow m^2+n^2+1+2mn-2m-2n \le m^2-2mn+n^2\\ \Leftrightarrow 4mn-2m-2n+1\le 0\\ \Leftrightarrow (2m-1)(2n-1)\le 0\ (2)$$
Mặt khác theo giả thiết, ta có $m,n\in \mathbb{N}^*\Rightarrow 2m-1,2n-1\ge 1$ nên ta có: $(2m-1)(2n-1)\ge 1\ (3)$

-Từ $(2)$ và $(3)$ thấy mâu thuẫn, vậy không tồn tại giá trị của $m,n$ thỏa mãn điều kiện bài toán (???)
---------------
Sao chả thấy ra ĐPCM gì thế này :( , sai ở đâu nhỉ :P
P/s: Lâu lâu không động đến số là đã thấy ngu ngu ="='


Chỗ $(m+n-1)^2|(m-n)^2\Rightarrow (m+n-1)^2\leq (m-n)^2$ là thiếu trường hợp $(m-n)^2=0$
Nó bằng 0 thì cái $(m+n-1)^2=10^{100}$ cũng k là đối :D